LSAT and Law School Admissions Forum

Get expert LSAT preparation and law school admissions advice from PowerScore Test Preparation.

 Administrator
PowerScore Staff
  • PowerScore Staff
  • Posts: 8916
  • Joined: Feb 02, 2011
|
#23058
Complete Question Explanation

Assumption-CE. The correct answer choice is (E)

Again, the author has left huge logical gaps in leaping to the conclusion about bronchial inhalers causing a higher asthma death rate. The question stem asks for a necessary assumption. Note that the correct answer choice will probably not make the argument perfectly air-tight; rather, it will just fill in one of the gaps.

Answer choice (A): This answer choice is not necessary for the argument. Apply the Assumption Negation test: Take the logical opposite of this statement and plug it back into the stimulus. What if urban pollution has doubled in the past decade? This actually would not destroy the author's argument. It is possible that urban pollution has doubled overall, but this is not causally instrumental in causing increased death rates from asthma. (Indeed, as the author points out, asthma death rates have increased even in cities with little or no urban pollution.)

Answer choice (B): First, we should be skeptical because the stimulus does not discuss allergies anywhere. Note that this answer choice is not saying, "Allergies are not the actual cause of increased asthma death rate." That would certainly be a necessary assumption, because it dismisses a possible alternative explanation for the observed increased death rate.

Answer choice (C): The naive approach might be to apply the Assumption Negation test and say to yourself, If bronchial inhalers were safe, then how could they ever lead to death? This might at first glance look like a necessary assumption. However, choice (D) of question 28 suggests one pathway by which bronchial inhalers, even if intrinsically safe per se, could lead to increased asthma death rates. So non-safety of inhalers is not necessary for the author's conclusion.

Answer choice (D): This is not a necessary assumption. If we made this assumption, it actually would not support the conclusion as specifically phrased (explaining the increased death rate from asthma). This assumption explains why increased deaths might happen (from other diseases) among asthma sufferers, but that is not the same as increased death rates from asthma. If we do not make this assumption, there are still other pathways by which bronchial inhalers could lead to asthma death.

Answer choice (E): This is the correct answer choice. This assumption is absolutely necessary. The author dismissed two possible explanations, then immediately concluded that a third explanation must be correct. In order to make this logical leap, we must assume that no other possible explanation exists.
 AspiringLawyer
  • Posts: 11
  • Joined: Oct 07, 2017
|
#40371
I agree and understand why E is the correct answer choice. On any given LSAT question, the stimulus exists in a vacuum. What is explicitly mentioned are the only possible causes. However, I do not fully understand why A is incorrect.

Answer Choice A: Urban pollution has not doubled in the past decade.
Assumption Negation Technique: Urban pollution has doubled in the past decade.
Inference from ANT: This statement strengthens the second of two “possible explanations” that the author disagrees with. Therefore, it does weaken the author’s conclusion that “one must instead conclude the cause of increased deaths is the use of bronchial inhalers by asthma sufferers.” In other words, this answer choice supports the cause that the author denies and disproves ("rate of deaths due to asthma has increased dramatically even in cities with... little or no urban pollution"). If increase in urban pollution is the cause, then use of bronchial inhalers cannot be the cause.

Can you kindly provide clarity?
 Francis O'Rourke
PowerScore Staff
  • PowerScore Staff
  • Posts: 471
  • Joined: Mar 10, 2017
|
#40385
In choosing answer choice (A) you may have overlooked the fact that cities with little or no urban pollution have seen a dramatic rise in the rates of deaths from asthma. Even if urban pollution were to have doubled everywhere, there would still have been cities with "little or no" pollution today and with skyrocketing rates of asthma deaths. In this case, the argument that "urban pollution is not to blame" would still stand, since we cannot account for the increased rates in the unpolluted or hardly polluted cities.

In short, the author did rule out both possible explanations mentioned at the beginning of the stimulus. The jump in logic was that once we rule out these two possible explanations, only one further explanation more remains. Answer choice (E) fills in this gap as the speaker assumed.
 Etsevdos
  • Posts: 62
  • Joined: Oct 22, 2017
|
#40904
Still struggling with C. Brought it down btwn E and C.

If the inhalers are safe why can we not presume that their effects are are also safe. D suggests to me that it is not necessarily the inhalers that are not safe; rather it is the effects of a separate issue that may be related to their use - I find it a bit circular. How can something be safe yet lead to potential death?
User avatar
 Jonathan Evans
PowerScore Staff
  • PowerScore Staff
  • Posts: 726
  • Joined: Jun 09, 2016
|
#40998
Hi, Etsevdos,

Good questions. Just to be clear, you indicate that you find (D) circular. It is not in fact circular but specious; it adds additional unnecessary suppositions into the argument. The information in answer choice (D) muddies up the argument, since it both contradicts the premise that there has been an increase in asthma deaths but perhaps bolsters the conclusion that the inhalers could be responsible for an increase in overall deaths. In any event, whether (D) strengthens, weakens, or does nothing at all to the argument is irrelevant because it is certainly not an "assumption on which the argument depends."

Let's briefly revisit this fundamental concept—"assumption on which the argument depends"—to remind ourselves that we are looking for a rather particular kind of answer choice, one that contains a necessary but unstated belief, the truth of which is required for the argument to be valid. In other words, this is not a strengthen question. It is an assumption question in which we seek something so fundamental and essential to the argument that if you took it away, the whole argument would fall apart.

With this in mind, let's remain focused on the conclusion: The cause of increased [asthma] deaths is the use of bronchial inhalers.

In our analysis, we must note that we're dealing with causal reasoning. The purported effect is the deaths. The purported cause is the inhalers.

In the author's other premises, he attempts to rule out two other possible causes: increased air pollution and increased reporting of asthma. He thereby reasons that only the inhalers could be to blame. Perhaps in your analysis you might ask, "Is there anything else that could have caused the deaths? How do we know it was the inhalers? The author certainly thinks it was the inhalers, but I'm less sure."

Thus, as you approach the answer choices, you should be equipped with these three observations:
  1. I'm looking for an essential, unstated belief of the author.
  2. The conclusion is that the inhalers must be causing the deaths.
  3. This is a causal argument. It's hard to prove that there is only one possible cause for an effect.
Now, let's look at (C) and (E).

Answer Choice (C): Must the author believe bronchial inhalers are unsafe, even when used according to instructions? Is this belief essential to the conclusion that the inhalers are causing the deaths? What if the inhalers are safe when used according to instructions? Could they still be responsible for the deaths? I guess so (this is the Assumption Negation Test™). Maybe the conclusion isn't as good. Maybe the information in (C) strengthens the argument, but is it really essential? No.

Answer Choice (E): Must the author believe that there is no other possible cause of the deaths other than pollution, inhalers, or reporting? Well, according to the premises, these three things certainly appear to be the only possibilities this author is willing to entertain. What if there were something else the author hasn't considered that could be causing the deaths? In this case, would the author's conclusion make any sense at all? Does it have to be the inhalers? No, if there were some other possible cause, then why would I think it's the inhalers? I wouldn't. The argument falls apart. This is a necessary assumption and the correct answer.

I hope this helps!
 stevendoering
  • Posts: 6
  • Joined: Oct 17, 2020
|
#80079
E is the correct answer. Mind you the question asks what assumption the ARGUMENT depends on. Upon breaking down the stimulus, we see a gap, namely that the author just assumes (without warrant) that because two proposed theories have been debunked, a third MUST be true.

C caught my eye at first. It would certainly help the authors conclusion, but the argument does not really depend on this being true. For example, C could be false (they ARE safe), and those using the inhalers go to the hospital/take medication/etc less due to the false sense of security.

Another reason (in addition to the comments on this forum) why A is wrong is due to the fact it brings up an argument that was already addressed in the stimulus (that even in cities with little/no pollution, the death rate has grown). Important also to not read "pollution" as "population".
 Adam Tyson
PowerScore Staff
  • PowerScore Staff
  • Posts: 5153
  • Joined: Apr 14, 2011
|
#81233
Good work, stevendoering! Nothing to add here, you've covered it all. The author says that in some cases the cause cannot be A or B and therefore must be C, and that argument assumes that there can be no cause other than A, B, or C. If there is some other possible cause, the argument falls apart. Nice job!

Get the most out of your LSAT Prep Plus subscription.

Analyze and track your performance with our Testing and Analytics Package.